Difference between revisions of "2000 AMC 10 Problems/Problem 17"

(New page: ==Problem== ==Solution== ==See Also== {{AMC10 box|year=2000|num-b=16|num-a=18}})
 
(Problem)
Line 1: Line 1:
 
==Problem==
 
==Problem==
 +
 +
Boris has an incredible coin changing machine. When he puts in a quarter, it returns five nickels; when he puts in a nickel, it returns five pennies; and when he puts in a penny, it returns five quarters. Boris starts with just one penny. Which of the following amounts could Boris have after using the machine repeatedly?
 +
 +
<math>\mathrm{(A)}</math> <dollar/><math>3.63</math>
 +
 +
<math>\mathrm{(B)}</math> <dollar/><math>5.13</math>
 +
 +
<math>\mathrm{(C)}</math> <dollar/><math>6.30</math>
 +
 +
<math>\mathrm{(D)}</math> <dollar/><math>7.45</math>
 +
 +
<math>\mathrm{(E)}</math> <dollar/><math>9.07</math>
  
 
==Solution==
 
==Solution==

Revision as of 08:36, 11 January 2009

Problem

Boris has an incredible coin changing machine. When he puts in a quarter, it returns five nickels; when he puts in a nickel, it returns five pennies; and when he puts in a penny, it returns five quarters. Boris starts with just one penny. Which of the following amounts could Boris have after using the machine repeatedly?

$\mathrm{(A)}$ <dollar/>$3.63$

$\mathrm{(B)}$ <dollar/>$5.13$

$\mathrm{(C)}$ <dollar/>$6.30$

$\mathrm{(D)}$ <dollar/>$7.45$

$\mathrm{(E)}$ <dollar/>$9.07$

Solution

See Also

2000 AMC 10 (ProblemsAnswer KeyResources)
Preceded by
Problem 16
Followed by
Problem 18
1 2 3 4 5 6 7 8 9 10 11 12 13 14 15 16 17 18 19 20 21 22 23 24 25
All AMC 10 Problems and Solutions